Chapter 19 – Performance and Remedies

Previous Chapter
Table of Contents
Next Chapter


LEARNING OBJECTIVES

After reading this chapter, you should understand the following:

  1. What performance is expected of the seller in a sales contract
  2. What performance is expected of the buyer in a sales contract
  3. What rights and duties the buyer has if there is a nonconforming delivery
  4. How, in general, the UCC approaches remedies
  5. What the seller’s remedies are for breach by the buyer
  6. What the buyer’s remedies are for breach by the seller
  7. What excuses the UCC provides for nonperformance

In Part II, we examined contract performance and remedies under common law. In this chapter, we examine performance and remedies under Article 2, the law of sales, of the Uniform Commercial Code (UCC). In the next chapter, we cover special remedies for those damaged or injured by defective products.

The parties often set out in their contracts the details of performance. These include price terms and terms of delivery—where the goods are to be delivered, when, and how. If the parties fail to list these terms, the rules studied in this chapter will determine the parties’ obligations: the parties may agree; if they do not, the UCC rules kick in as the default. In any event, the parties have an obligation to act in good faith.

19.1 Performance by the Seller

LEARNING OBJECTIVE

  1. Understand what is meant when it is said the seller has a duty to “make a timely delivery of conforming goods.”

The Seller’s Duty in General

The general duty of the seller is this: to make a timely delivery of conforming goods.Uniform Commercial Code, Sections 2-301and 2-309.

The CISG, Article 30, says, “The seller must deliver the goods, hand over any documents relating to them and transfer the property in the goods, as required by the contract and this Convention.”

Analysis of the Seller’s Duty

Timing

By agreement or stipulation, the parties may fix the time when delivery is to be made by including statements in contracts such as “Delivery is due on or before July 8” or “The first of 12 installments is due on or before July 8.” Both statements are clear.

If the parties do not stipulate in their contract when delivery is to occur, the UCC fills the gap. Section 2-309 of the UCC says, “The time for shipment or any other action under a contract if not provided for in this Article or agreed upon shall be a reasonable time.” And what is a “reasonable time” is addressed by comment 1 to this section:

It thus turns on the criteria as to “reasonable time” and on good faith and commercial standards set forth in Sections 1-202, 1-203 and 2-103. It…depends on what constitutes acceptable commercial conduct in view of the nature, purposes and circumstances of the action to be taken.

The CISG (Article 33) provides as follows:

The seller must deliver the goods

(a) if a date is fixed by or determinable from the contract, on that date;

(b) if a period of time is fixed by or determinable from the contract, at any time within that period unless circumstances indicate that the buyer is to choose a date; or

(c) in any other case, within a reasonable time after the conclusion of the contract.

Delivery

The parties may agree as to how delivery shall be accomplished; if they do not, the UCC fills the gap.

The CISG (Article 31) says this:

If the seller is not bound to deliver the goods at any other particular place, his obligation to deliver consists

(a) if the contract of sale involves carriage of the goods—in handing the goods over to the first carrier for transmission to the buyer;

(b) if, in cases not within the preceding subparagraph…in placing the goods at the buyer’s disposal at that place [where the goods are];

(c) in other cases—in placing the goods at the buyer’s disposal at the place where the seller had his place of business at the time of the conclusion of the contract.

By Agreement

The parties may use any language they want to agree on delivery terms.

If There Is No Agreement

If the parties do not stipulate delivery terms or if their agreement is incomplete or merely formulaic, the UCC describes the seller’s obligations or gives meaning to the formulaic language. (Because form contracts are prevalent, formulaic language is customary.) You recall the discussion in Chapter 18 "Title and Risk of Loss" about when title shifts: we said title shifts when the seller has completed delivery obligations under the contract, and we ran through how those obligations are usually expressed. A quick review here is appropriate.

The contract may be either a shipment contract, a destination contract, or a contract where the goods are not to be moved (being held by a bailee). In any case, unless otherwise agreed, the delivery must be at a reasonable time and the tender (the offer to make delivery) must be kept open for a reasonable time; the buyer must furnish facilities “reasonably suited to the receipt of the goods.”Uniform Commercial Code, Section 2-503.

In a shipment contract, the seller has four duties: (1) to deliver the goods to a carrier; (2) to deliver the goods with a reasonable contract for their transportation; (3) to deliver them with proper documentation for the buyer; and (4) to promptly notify the buyer of the shipment (UCC, Section 2-504). The contract may set out the seller’s duties using customary abbreviations, and the UCC interprets those: “F.O.B [insert place where goods are to be shipped from]” means “free on board”—the seller must see to it that the goods are loaded on the vehicle of conveyance at the place of shipment. “F.A.S. [port of shipment inserted here]” means the seller must see to it that the goods are placed along the ship on the dock ready to be loaded (Section 2-319). Price terms include “C.I.F.,” which means the sale price includes the cost of the goods, insurance, and freight charges, and “C. & F.,” which means the sales price includes the cost of the goods at a cheaper unit price and freight but not insurance.Uniform Commercial Code, Section 2-320. If it is clear from the contract that the seller is supposed to ship the goods (i.e., the buyer is not going to the seller’s place to get them) but not clear whether it is a shipment or a destination contract, the UCC presumes it is a shipment contract.Uniform Commercial Code, Section 2-503(5).

If it is a destination contract, the seller has two duties: to get the goods to the destination at the buyer’s disposal and to provide appropriate documents of delivery.Uniform Commercial Code, Section 2-503. The contract language could be “F.O.B. [place of destination inserted here],” which obligates the seller to deliver to that specific location; “ex-ship,” which obligates the seller to unload the goods from the vehicle of transportation at the agreed location (e.g., load the goods onto the dock); or it could be “no arrival, no sale,” where the seller is not liable for failure of the goods to arrive, unless she caused it.Uniform Commercial Code, Sections 2-319, 2-322, and 2-324.

If the goods are in the possession of a bailee and are not to be moved—and the parties don’t stipulate otherwise—the UCC, Section 2-503 says delivery is accomplished when the seller gives the buyer a negotiable document of title, or if none, when the bailee acknowledges the buyer’s right to take the goods.

If nothing at all is said about delivery, the place for delivery is the seller’s place of business or his residence if he has no place of business.Uniform Commercial Code, Section 2-308.

Conforming Goods

As always, the parties may put into the contract whatever they want about the goods as delivered. If they don’t, the UCC fills the gaps.

By Agreement

The parties may agree on what “conforming goods” means. An order will specify “large grade A eggs,” and that means something in the trade. Or an order might specify “20 gross 100-count boxes No. 8 × 3/8 × 32 Phillips flathead machine screws.” That is a screw with a designated diameter, length, number of threads per inch, and with a unique, cruciform head insert to take a particular kind of driver. The buyer might, for example, agree to purchase “seconds,” which are goods with some flaw, such as clothes with seams not sewed quite straight or foodstuffs past their pull date. The parties may also agree in the contract what happens if nonconforming goods are delivered, as we’ll see later in this chapter.

If There Is No Agreement

If nothing is said in the contract about what quality of goods conform to the contract, then the UCC default rule kicks in. The seller is to make a perfect tender: what is delivered must in every respect conform to the contract.Uniform Commercial Code, Section 2-601. And if what is delivered doesn’t conform to the contract, the buyer is not obligated to accept the goods.

The CISG has no perfect tender rule. Article 46 provides this:

If the goods do not conform with the contract, the buyer may require delivery of substitute goods only if the lack of conformity constitutes a fundamental breach of contract and a request for substitute goods is made either in conjunction with notice given under article 39 or within a reasonable time thereafter. If the goods do not conform with the contract, the buyer may require the seller to remedy the lack of conformity by repair, unless this is unreasonable having regard to all the circumstances. A request for repair must be made either in conjunction with notice given under article 39 or within a reasonable time thereafter.

Installment Contracts

Unless otherwise agreed, all goods should be delivered at one time, and no payment is due until tender. But where circumstances permit either party to make or demand delivery in lots, Section 2-307 of the UCC permits the seller to demand payment for each lot if it is feasible to apportion the price. What if the contract calls for delivery in installment, and one installment is defective—is that a material breach of the whole contract? No. Section 2-612 of the UCC says this:

(2) The buyer may reject any installment which is non-conforming if the non-conformity substantially impairs the value of that installment and cannot be cured or if the non-conformity is a defect in the required documents; but if the non-conformity does not fall within subsection (3) and the seller gives adequate assurance of its cure the buyer must accept that installment.

(3) Whenever non-conformity or default with respect to one or more installments substantially impairs the value of the whole contract there is a breach of the whole.

Cure for Improper Delivery

Failure to make a perfect tender, unless otherwise agreed, is a material breach of the sales contract. However, before the defaulting seller is in complete default, she has a right to cure. Here’s what the UCC says in Section 2-508:

(1) Where any tender or delivery by the seller is rejected because non-conforming and the time for performance has not yet expired, the seller may seasonably notify the buyer of his intention to cure and may then within the contract time make a conforming delivery.

(2) Where the buyer rejects a non-conforming tender which the seller had reasonable grounds to believe would be acceptable with or without money allowance the seller may if he seasonably notifies the buyer have a further reasonable time to substitute a conforming tender.

Buyer orders Santa Claus candles deliverable November 5; on October 25 the goods are delivered, but they’re not right: they’re Christmas angel candles instead. But the seller still has eleven days to cure, and the buyer must allow that. Buyer places an order exactly the same as the first order, and the order arrives on November 5 in the original manufacturer’s packaging, but they’re not right. “Well,” says the seller, “I thought they’d be OK right out of the package. I’ll get the correct ones to you right away.” And the buyer would have a duty to allow that, if “right away” is a “further reasonable time.”

Article 48 of the CISG says this:

The seller may, even after the date for delivery, remedy at his own expense any failure to perform his obligations, if he can do so without unreasonable delay and without causing the buyer unreasonable inconvenience or uncertainty of reimbursement by the seller of expenses advanced by the buyer. However, the buyer retains any right to claim damages as provided for in this Convention. If the seller requests the buyer to make known whether he will accept performance and the buyer does not comply with the request within a reasonable time, the seller may perform within the time indicated in his request. The buyer may not, during that period of time, resort to any remedy which is inconsistent with performance by the seller.

So, again, the seller’s duty is to make a timely delivery of conforming goods. Let’s take a look now at the buyer’s duties.

KEY TAKEAWAY

The seller’s obligation under the UCC is to make a timely delivery of conforming goods. For each element of the duty—timely, delivery, conforming goods—the parties may agree in their contract. If they do not, the UCC fills in default rules.

EXERCISES

  1. If the parties do not specify a time for delivery, what is the UCC’s default position?
  2. What are the seller’s obligations in an F.O.B. shipment contract? In an F.O.B. destination contract?
  3. Compare the UCC’s perfect tender rule to the common-law substantial performance doctrine.

19.2 Performance by Buyer

LEARNING OBJECTIVES

  1. Understand what the general duties of the buyer are.
  2. Recognize what rights the buyer has if the seller tenders a nonconforming delivery.

General Duties of Buyer

The general duty of the buyer is this: inspection, acceptance, and payment.Uniform Commercial Code, Sections 2-301 and 2-513. But the buyer’s duty does not arise unless the seller tenders delivery.

Inspection

Under Sections 2-513(1) and (2) of the Uniform Commercial Code (UCC), the buyer has a qualified right to inspect goods. That means the buyer must be given the chance to look over the goods to determine whether they conform to the contract. If they do not, he may properly reject the goods and refuse to pay. The right to inspect is subject to three exceptions:

  1. The buyer waives the right. If the parties agree that payment must be made before inspection, then the buyer must pay (unless the nonconformity is obvious without inspection). Payment under these circumstances does not constitute acceptance, and the buyer does not lose the right to inspect and reject later.
  2. The delivery is to be made C.O.D. (cash on delivery).
  3. Payment is to be made against documents of title.

If the buyer fails to inspect, or fails to discover a defect that an inspection would have revealed, he cannot later revoke his acceptance, subject to some exceptions.

Acceptance

Acceptance is clear enough: it means the buyer takes the goods. But the buyer’s options on improper delivery need to be examined, because that’s often a problem area.

The buyer may accept goods by words, silence, or action. Section 2-606(1) of the UCC defines acceptance as occurring in any one of three circumstances:

  1. Words. The buyer, after a reasonable opportunity to inspect, tells the seller either that the goods conform or that he will keep them despite any nonconformity.
  2. Silence. The buyer fails to reject, after a reasonable opportunity to inspect.
  3. Action. The buyer does anything that is inconsistent with the seller’s ownership, such as using the goods (with some exceptions) or selling the goods to someone else.

Once the buyer accepts, she is obligated to pay at the contract rate and loses the right to reject the goods.Uniform Commercial Code, Section 2-607. She is stuck, subject to some exceptions.

Payment

The parties may specify in their contract what payment means and when it is to be made. If they don’t, the UCC controls the transaction.Uniform Commercial Code, Sections 2-511 and 2-512.

A Buyer’s Right on Nonconforming Delivery

Obviously if the delivery is defective, the disappointed buyer does not have to accept the goods: the buyer may (a) reject the whole, (b) accept the whole, or (c) accept any commercial unit and reject the rest (2-601, 2A-509), or (d)—in two situations—revoke an acceptance already made.

Rejection and a Buyer’s Duties after Rejection

Under UCC, Section 2-601(a), rejection is allowed if the seller fails to make a perfect tender. The rejection must be made within a reasonable time after delivery or tender. Once it is made, the buyer may not act as the owner of the goods. If he has taken possession of the goods before he rejects them, he must hold them with reasonable care to permit the seller to remove them. If the buyer is a merchant, then the buyer has a special duty to follow reasonable instructions from the seller for disposing of the rejected goods; if no instructions are forthcoming and the goods are perishable, then he must try to sell the goods for the seller’s account and is entitled to a commission for his efforts. Whether or not he is a merchant, a buyer may store the goods, reship them to the seller, or resell them—and charge the seller for his services—if the seller fails to send instructions on the goods’ disposition. Such storage, reshipping, and reselling are not acceptance or conversion by the buyer.

Acceptance of a Nonconforming Delivery

The buyer need not reject a nonconforming delivery. She may accept it with or without allowance for the nonconformity.

Acceptance of Part of a Nonconforming Delivery

The buyer may accept any commercial unit and reject the rest if she wants to. A commercial unit means “such a unit of goods as by commercial usage is a single whole for purposes of sale and division of which materially impairs its character or value on the market or in use. A commercial unit may be a single article (as a machine), a set of articles (as a suite of furniture or an assortment of sizes), a quantity (as a bale, gross, or carload), or any other unit treated in use or in the relevant market as a single whole.”Uniform Commercial Code, Sections 2-105 and 2A103(1).

Installment Sales

A contract for an installment sale complicates the answer to the question, “What right does the buyer have to accept or reject when the seller fails to deliver properly?” (An installment contract is one calling for delivery of goods in separate lots with separate acceptance for each delivery.) The general answer is found in the UCC at Section 2-612, which permits the buyer to reject any nonconforming installment if the nonconformity cannot be cured if it substantially impairs the value of that particular installment. However, the seller may avoid rejection by giving the buyer adequate assurances that he will cure the defect, unless the particular defect substantially impairs the value of the whole contract.

Suppose the Corner Gas Station contracts to buy 12,000 gallons of regular gasoline from Gasoline Seller, deliverable in twelve monthly installments of 1,000 gallons on the first of each month, with a set price payable three days after delivery. In the third month, Seller is short and can deliver only 500 gallons immediately and will not have the second 500 gallons until midmonth. May Corner Gas reject this tender? The answer depends on the circumstances. The nonconformity clearly cannot be cured, since the contract calls for the full 1,000 on a particular day. But the failure to make full delivery does not necessarily impair the value of that installment; for example, Corner Gas may know that it will not use up the 500 gallons until midmonth. However, if the failure will leave Corner Gas short before midmonth and unable to buy from another supplier unless it agrees to take a full 1,000 (more than it could hold at once if it also took Seller’s 500 gallons), then Corner Gas is entitled to reject Seller’s tender.

Is Corner Gas entitled to reject the entire contract on the grounds that the failure to deliver impairs the value of the contract as a whole? Again, the answer depends on whether the impairment was substantial. Suppose other suppliers are willing to sell only if Corner Gas agrees to buy for a year. If Corner Gas needed the extra gasoline right away, the contract would have been breached as whole, and Corner Gas would be justified in rejecting all further attempted tenders of delivery from Seller. Likewise, if the spot price of gasoline were rising so that month-to-month purchases from other suppliers might cost it more than the original agreed price with Seller, Corner Gas would be justified in rejecting further deliveries from Seller and fixing its costs with a supply contract from someone else. Of course, Corner Gas would have a claim against Seller for the difference between the original contract price and what it had to pay another supplier in a rising market (as you’ll see later in this chapter).

Revocation

revocation of acceptance means that although the buyer has accepted and exercised ownership of the goods, he can return the goods and get his money back. There are two circumstances in which the buyer can revoke an acceptance if the nonconformity “substantially impairs its value to him”:Uniform Commercial Code, Section 2-608.

  1. if the buyer reasonably thought the nonconformity would be cured and it is not within a reasonable time; or
  2. if the acceptance was due to a latent defect that could not reasonably have been discovered before acceptance.

Consider two examples illustrated in the next paragraph. The first deals with point a (buyer thought nonconformity would be cured and it was not within a reasonable time), and the second gets to point b (latent defect).

In August 1983, the Borsages purchased a furnished mobile home on the salesperson’s assertion that it was “the Cadillac of mobile homes.” But when they moved in, the Borsages discovered defects: water leaks, loose moldings, a warped dishwasher door, a warped bathroom door, holes in walls, defective heating and cooling systems, cabinets with chips and holes, furniture that fell apart, mold and mildew in some rooms, a closet that leaked rainwater, and defective doors and windows. They had not seen these defects at the time of purchase because they looked at the mobile home at night and there were no lights on in it. The Borsages immediately complained. Repairmen came by but left, only promising to return again. Others did an inadequate repair job by cutting a hole in the bottom of the home and taping up the hole with masking tape that soon failed, causing the underside of the home to pooch out. Yet more repairmen came by but made things worse by inadvertently poking a hole in the septic line and failing to fix it, resulting in a permanent stench. More repairmen came by, but they simply left a new dishwasher door and countertop at the home, saying they didn’t have time to make the repairs. In June 1984, the Borsages provided the seller a long list of uncorrected problems; in October they stopped making payments. Nothing happened. In March 1986—thirty-one months after buying the mobile home—they told the seller to pick up the mobile home: they revoked their acceptance and sued for the purchase price. The defendant seller argued that the Borsages’ failure to move out of the house for so long constituted acceptance. But they were repeatedly assured the problems would be fixed, and moreover they had no place else to live, and no property to put another mobile home on if they abandoned the one they had. The court had no problem validating the Borsages’ revocation of acceptance, under the section noted earlier, if they ever had accepted it. The seller might have a right to some rental value, though.North River Homes, Inc., v. Borsage, Mississippi (1992).

In April 1976, Clarence Miller ordered a new 1976 Dodge Royal Monaco station wagon from plaintiff Colonial Dodge. The car included a heavy-duty trailer package with wide tires. The evening of the day the Millers picked up the new car, Mrs. Miller noticed that there was no spare tire. The following morning, the defendant notified the plaintiff that he insisted on a spare tire, but when he was told there were no spare tires available (because of a labor strike), Mr. Miller told the plaintiff’s salesman that he would stop payment on the check he’d given them and that the car could be picked up in front of his house. He parked it there, where it remained until the temporary registration sticker expired and it was towed by the police to an impound yard. Plaintiff sued for the purchase price, asserting that the missing spare tire did not “substantially impair the value of the goods to the buyer.” On appeal to the Michigan Supreme Court, the plaintiff lost. “In this case the defendant’s concern with safety is evidenced by the fact that he ordered the special package which included spare tires. The defendant’s occupation demanded that he travel extensively, sometimes in excess of 150 miles per day on Detroit freeways, often in the early morning hours.…He was afraid of a tire going flat…at 3 a.m. Without a spare, he would be helpless until morning business hours. The dangers attendant upon a stranded motorist are common knowledge, and Mr. Miller’s fears are not unreasonable.” The court observed that although he had accepted the car before he discovered the nonconformity, that did not preclude revocation: the spare was under a fastened panel, concealed from view.Colonial Dodge v. Miller, 362 N.W.2d 704 (Mich. 1984).

KEY TAKEAWAY

The duty of the buyer in a sales contract is to inspect, accept, and pay. Failure to discover a defect that an inspection would have revealed is a waiver of right to complain. Normally the goods are conforming and the buyer accepts them, but upon discovery of a defect the buyer may reject the whole nonconforming delivery, part of it (the buyer has some duties if she has possession of the rejected goods), or in some cases reject one installment of an installment sale or, if one defective installment is serious enough to vitiate the whole contract, the buyer may consider the contract terminated. If goods have been accepted because the seller promised to fix defects or because the defects were latent, then the buyer may revoke the acceptance where the nonconformity substantially impairs the value of the contract to the buyer.

EXERCISES

  1. If a buyer takes possession of goods and shortly thereafter discovers they are nonconforming, what duty does the nonmerchant buyer have with respect to the goods? What duty does the merchant buyer have with respect to the goods?
  2. What is the difference between rejection and revocation?
  3. Under what circumstances will a defective installment allow the buyer to reject that installment? Under what circumstances would a defective installment allow the buyer to terminate the contract?

19.3 Remedies

LEARNING OBJECTIVES

  1. Understand what purpose remedies serve under the UCC.
  2. Be able to see when the parties’ agreements as to limited remedies fail under the UCC.
  3. Recognize what the seller’s remedies are.
  4. Recognize what the buyer’s remedies are.

Remedies in General

General Policy

The general policy of the Uniform Commercial Code (UCC) is to put the aggrieved party in a good position as if the other party had fully performed—as if there had been a timely delivery of conforming goods. The UCC provisions are to be read liberally to achieve that result if possible. Thus the seller has a number of potential remedies when the buyer breaches, and likewise the buyer has a number of remedies when the seller breaches.

The CISG provides, at Article 74:

Damages for breach of contract by one party consist of a sum equal to the loss, including loss of profit, suffered by the other party as a consequence of the breach. Such damages may not exceed the loss which the party in breach foresaw or ought to have foreseen at the time of the conclusion of the contract, in the light of the facts and matters of which he then knew or ought to have known, as a possible consequence of the breach of contract.

Specifying Remedies

We have emphasized how the UCC allows people to make almost any contract they want (as long as it’s not unconscionable). Just as the parties may specify details of performance in the contract, so they may provide for and limit remedies in the event of breach.Uniform Commercial Code, Sections 2-719(1) and 2A-503(1). The following would be a typical limitation of remedy: “Seller’s sole obligation in the event goods are deemed defective by the seller is to replace a like quantity of nondefective goods.” A remedy is optional unless it is expressly agreed that it is the exclusive remedy.Uniform Commercial Code, Sections 2-719(1)(b) and 2A-503(2).

But the parties are not free to eliminate all remedies. As the UCC comment to this provision puts it, “If the parties intend to conclude a contract for sale within this Article they must accept the legal consequence that there be at least a fair quantum of remedy for breach of the obligations or duties outlined in the contract.” In particular, the UCC lists three exemptions from the general rule that the parties are free to make their contract up any way they want as regards remedies:

  1. When the circumstances cause the agreed-to remedy to fail or be ineffective, the default UCC remedy regime works instead.Uniform Commercial Code, Sections 2-719(2) and 2A-503(2).
  2. Consequential damages may be limited or excluded unless the limitation or exclusion is unconscionable. Limitation of consequential damages for injury to the person in the case of consumer goods is prima facie unconscionable, but limitation of damages where the loss is commercial is not.Uniform Commercial Code, Sections 2-719(3) and 2A-503(2).
  3. The parties may agree to liquidated damages: “Damages for breach by either party may be liquidated in the agreement but only at an amount which is reasonable in the light of the anticipated or actual harm caused by the breach, the difficulties of proof of loss, and the inconvenience or nonfeasibility of otherwise obtaining an adequate remedy. A term fixing unreasonably large liquidated damages is void as a penalty.”Uniform Commercial Code, Section 2-718. The Code’s equivalent position on leases is interestingly slightly different. UCC 2A-504(1) says damages may be liquidated “but only at an amount or by a formula that is reasonable in light of the then anticipated harm caused” by the breach. It leaves out anything about difficulties of proof or inconvenience of obtaining another adequate remedy.

Statute of Limitations

The UCC statute of limitations for breach of any sales contract is four years. The parties may “reduce the period of limitation to not less than one year but may not extend it.”Uniform Commercial Code, Section 2-725. Article 2A-506(1) is similar, but omits the prohibition against extending the limitation. Article 2-725(2) goes on: “A cause of action accrues when the breach occurs, regardless of the aggrieved party’s lack of knowledge of the breach. A breach of warranty occurs when tender of delivery is made, except that where a warranty explicitly extends to future performance of the goods and discovery of the breach must await the time of such performance the cause of action accrues when the breach is or should have been discovered.”

Article 2A-506(2) is similar to 2-725(2).

Seller’s Remedies

Article 2 in General

Article 2-703 of the UCC lists the four things the buyer can do by way of default, and it lists—here slightly paraphrased—the seller’s remedies (2A-523(1) is similar for leases):

Where the buyer wrongfully rejects or revokes acceptance of goods or fails to make a payment due on or before delivery or repudiates with respect to a part or the whole, then with respect to any goods directly affected and, if the breach is of the whole contract, then also with respect to the whole undelivered balance, the aggrieved seller may:

(1) withhold delivery of such goods;

(2) stop delivery by any bailee;

(3) identify to the contract conforming goods not already identified;

(4) reclaim the goods on the buyer’s insolvency;

(5) resell and recover damages;

(6) recover damages for non-acceptance or repudiation;

(7) or in a proper case recover the price;

(8) cancel.

Items (1)–(4) address the seller’s rights to deal with the goods; items (5)–(7) deal with the seller’s rights as regards the price, and item (8) deals with the continued existence of the contract.

The CISG’s take is similar. Article 61 and following state,

If the buyer fails to perform any of his obligations under the contract or this Convention, the seller may:…(a) require the buyer to pay the price. (b) Fix an additional period of time of reasonable length for performance by the buyer of his obligations; unless the seller has received notice from the buyer that he will not perform within the period so fixed, the seller may not, during that period, resort to any remedy for breach of contract. (c) Declare the contract avoided if the failure by the buyer to perform any of his obligations under the contract or this Convention amounts to a fundamental breach of contract or if the buyer does not, within the additional period of time fixed by the seller [above], perform his obligation to pay the price or take delivery of the goods, or if he declares that he will not do so within the period so fixed. (d) The seller also has the right to damages.

To illustrate the UCC’s remedy provision, in this and the following section, we assume these facts: Howard, of Los Angeles, enters into a contract to sell and ship one hundred prints of a Pieter Bruegel painting, plus the original, to Bunker in Dallas. Twenty-five prints have already been delivered to Bunker, another twenty-five are en route (having been shipped by common carrier), another twenty-five are finished but haven’t yet been shipped, and the final twenty-five are still in production. The original is hanging on the wall in Howard’s living room. We will take up the seller’s remedies if the buyer breaches and if the buyer is insolvent.

Remedies on Breach

Bunker, the buyer, breaches the contract. He sends Howard an e-mail stating that he won’t buy and will reject the goods if delivery is attempted. Howard has the following cumulative remedies; election is not required.

Withhold Further Delivery

Howard may refuse to send the third batch of twenty-five prints that are awaiting shipment.

Stop Delivery

Howard may also stop the shipment. If Bunker is insolvent, and Howard discovers it, Howard would be permitted to stop any shipment in the possession of a carrier or bailee. If Bunker is not insolvent, the UCC permits Howard to stop delivery only of carload, truckload, planeload, or larger shipment. The reason for limiting the right to bulk shipments in the case of noninsolvency is that stopping delivery burdens the carrier and requiring a truck, say, to stop and the driver to find a small part of the contents could pose a sizeable burden.

Identify to the Contract Goods in Possession

Howard could “identify to the contract” the twenty-five prints in his possession. Section 2-704(1) of the UCC permits the seller to denote conforming goods that were not originally specified as the exact objects of the contract, if they are under his control or in his possession at the time of the breach. Assume that Howard had five hundred prints of the Bruegel painting. The contract did not state which one hundred of those prints he was obligated to sell, but once Bunker breached, Howard could declare that those particular prints were the ones contemplated by the contract. He has this right whether or not the identified goods could be resold. Moreover, Howard may complete production of the twenty-five unfinished prints and identify them to the contract, too, if in his “reasonable commercial judgment” he could better avoid loss—for example, by reselling them. If continued production would be expensive and the chances of resale slight, the seller should cease manufacture and resell for scrap or salvage value.

Resell

Howard could resell the seventy-five prints still in his possession as well as the original. As long as he proceeds in good faith and in a commercially reasonable manner, per Section 2-706(2) and Section 2A-527(3), he is entitled to recover the difference between the resale price and the contract price, plus incidental damages (but less any expenses saved, like shipping expenses). “Incidental damages” include any reasonable charges or expenses incurred because, for example, delivery had to be stopped, new transportation arranged, storage provided for, and resale commissions agreed on.

The seller may resell the goods in virtually any way he desires as long as he acts reasonably. He may resell them through a public or private sale. If the resale is public—at auction—only identified goods can be sold, unless there is a market for a public sale of futures in the goods (as there is in agricultural commodities, for example). In a public resale, the seller must give the buyer notice unless the goods are perishable or threaten to decline in value speedily. The goods must be available for inspection before the resale, and the buyer must be allowed to bid or buy.

The seller may sell the goods item by item or as a unit. Although the goods must relate to the contract, it is not necessary for any or all of them to have exited or to have been identified at the time of breach.

The seller does not owe the buyer anything if resale or re-lease results in a profit for the buyer.Uniform Commercial Code, Sections 2-706 and 2A-527.

Recover Damages

The seller may recover damages equal to the difference between the market price (measured at the time and place for tender of delivery) and the unpaid contract price, plus incidental damages, but less any expenses saved because of the buyer’s breach. Suppose Howard’s contract price was $100 per print plus $10,000 for the original and that the market price on the day Howard was to deliver the seventy-five prints was $75 (plus $8,000 for the original). Suppose too that the shipping costs (including insurance) that Howard saved when Bunker repudiated were $2,000 and that to resell them Howard would have to spend another $750. His damages, then, would be calculated as follows: original contract price ($17,500) less market price ($13,625) = $3,875 less $2,000 in saved expenses = $1,875 plus $750 in additional expenses = $2,625 net damages recoverable by Howard, the seller.

The CISG puts it similarly in Article 75: “If the contract is avoided and if, in a reasonable manner and within a reasonable time after avoidance, the buyer has bought goods in replacement or the seller has resold the goods, the party claiming damages may recover the difference between the contract price and the price in the substitute transaction as well as any further damages recoverable.”

If the formula would not put the seller in as good a position as performance under the contract, then the measure of damages is lost profits—that is, the profit that Howard would have made had Bunker taken the original painting and prints at the contract price (again, deducting expenses saved and adding additional expenses incurred, as well as giving credit for proceeds of any resale).Uniform Commercial Code, Section 2-708(2); Section 2A-528(2) is similar. This provision becomes especially important for so-called lost volume sellers. Howard may be able to sell the remaining seventy-five prints easily and at the same price that Bunker had agreed to pay. Then why isn’t Howard whole? The reason is that the second buyer was not a substitute buyer but an additional one; that is, Howard would have made that sale even if Bunker had not reneged on the contract. So Howard is still short a sale and is out a profit that he would have made had Bunker honored the contract.

Recover the Price

Howard—the seller—could recover from Bunker for the price of the twenty-five prints that Bunker holds. Or suppose they had agreed to a shipment contract, so that the risk of loss passed to Bunker when Howard placed the other prints with the trucker and that the truck crashed en route and the cargo destroyed. Howard could recover the price. Or suppose there were no market for the remaining seventy-five prints and the original. Howard could identify these prints to the contract and recover the contract price. If Howard did resell some prints, the proceeds of the sale would have to be credited to Bunker’s account and deducted from any judgment. Unless sold, the prints must be held for Bunker and given to him upon his payment of the judgment.

Cancel the Contract

When Bunker repudiated, Howard could declare the contract cancelled. This would also apply if a buyer fails to make a payment due on or before delivery. Cancellation entitles the nonbreaching party to any remedies for the breach of the whole contract or for any unperformed balance. That is what happens when Howard recovers damages, lost profits, or the price.Uniform Commercial Code, Sections 2-703(f) and 2A-524(1)(a).

Again, the CISG is similar. Article 64 provides that the seller may declare the contract avoided “if the failure by the buyer to perform any of his obligations under the contract or this Convention amounts to a fundamental breach of contract; or if the buyer does not, within the additional period of time fixed by the seller perform his obligation to pay the price or take delivery of the goods, or if he declares that he will not do so within the period so fixed.”

Note again that these UCC remedies are cumulative. That is, Howard could withhold future delivery and stop delivery en route, and identify to the contract goods in his possession, and resell, and recover damages, and cancel.

Remedies on Insolvency

The remedies apply when the buyer breaches the contract. In addition to those remedies, the seller has remedies when he learns that the buyer is insolvent, even if the buyer has not breached. Insolvency results, for example, when the buyer has “ceased to pay his debts in the ordinary course of business,” or the buyer “cannot pay his debts as they become due.”Uniform Commercial Code, Section 1-201(23).

Upon learning of Bunker’s insolvency, Howard could refuse to deliver the remaining prints, unless Bunker pays cash not only for the remaining prints but for those already delivered. If Howard learned of Bunker’s insolvency within ten days of delivering the first twenty-five prints, he could make a demand to reclaim them. If within three months prior to delivery, Bunker had falsely represented that he was solvent, the ten-day limitation would not cut off Howard’s right to reclaim. If he does seek to reclaim, Howard will lose the right to any other remedy with respect to those particular items. However, Howard cannot reclaim goods already purchased from Bunker by a customer in the ordinary course of business. The customer does not risk losing her print purchased several weeks before Bunker has become insolvent.Uniform Commercial Code, Section 2-702 (3).

In the lease situation, of course, the goods belong to the lessor—the lessor has title to them—so the lessor can repossess them if the lessee defaults.Uniform Commercial Code, Section 2A-525(2).

Buyer’s Remedies

In this section, let us assume that Howard, rather than Bunker, breaches, and all other circumstances are the same. That is, Howard had delivered twenty-five prints, twenty-five more were en route, the original painting hung in Howard’s living room, another twenty-five prints were in Howard’s factory, and the final twenty-five prints were in production.

In General

The buyer can do the following three things by way of defaulting: repudiate the contract, fail to deliver the goods, or deliver or tender nonconforming goods. Section 2-711 of the UCC provides the following remedies for the buyer:

Where the seller fails to make delivery or repudiates, or the buyer rightfully rejects or justifiably revokes, then with respect to any goods involved, and with respect to the whole if the breach goes to the whole contract, the buyer may

(1) cancel the contract, and

(2) recover as much of the price as has been paid; and

(3) “cover” and get damages; and

(4) recover damages for nondelivery.

Where the seller fails to deliver or repudiates, the buyer may also:

(5) if the goods have been identified recover them; or

(6) in a proper case obtain specific performance or

(7) replevy the goods.

On rightful rejection or justifiable revocation of acceptance, a buyer:

(8) has a security interest in goods in his possession or control for any payments made on their price and any expenses reasonably incurred in their inspection, receipt, transportation, care and custody and may hold such goods and resell them in like manner as an aggrieved seller.

If the buyer has accepted non-conforming goods and notified seller of the non-conformity, buyer can

(9) recover damages for the breach;Uniform Commercial Code, Section 2-714.

and in addition the buyer may

(10) recover incidental damages and

(11) recover consequential damages.Uniform Commercial Code, Section 2-715.

Thus the buyer’s remedies can be divided into two general categories: (1) remedies for goods that the buyer does not receive or accept, when he has justifiably revoked acceptance or when the seller repudiates, and (2) remedies for goods accepted.

The CISG provides similar remedies at Articles 45–51:

If the seller fails to perform any of his obligations under the contract, buyer may (1) declare the contract avoided if the seller’s breach is fundamental; or (2) require performance by the seller of his obligations unless the buyer has resorted to a remedy which is inconsistent with this requirement; (3) require delivery of substitute goods if the non-conformity constitutes a fundamental breach of contract; (4) may require the seller to remedy the lack of conformity by repair, unless this is unreasonable having regard to all the circumstances; (5) may fix an additional period of time of reasonable length for performance by the seller of his obligations and unless the buyer has received notice from the seller that he will not perform within the period so fixed, the buyer may not, during that period, resort to any remedy for breach of contract; (6) in case of non-conforming delivery, reduce the price in the same proportion as the value that the goods actually delivered had at the time of the delivery bears to the value that conforming goods would have had at that time.

Goods Not Received

The UCC sets out buyer’s remedies if goods are not received or if they are rightfully rejected or acceptance is rightfully revoked.

Cancel

If the buyer has not yet received or accepted the goods (or has justifiably rejected or revoked acceptance because of their nonconformity), he may cancel the contract and—after giving notice of his cancellation—he is excused from further performance.Uniform Commercial Code, Sections 2-711(1), 2-106, 2A-508(1)(a), and 2A-505(1).

Recover the Price

Whether or not the buyer cancels, he is entitled to recover the price paid above the value of what was accepted.

Cover

In the example case, Bunker—the buyer—may “cover” and have damages: he may make a good-faith, reasonable purchase of substitute goods. He may then recover damages from the seller for the difference between the cost of cover and the contract price. This is the buyer’s equivalent of the seller’s right to resell. Thus Bunker could try to purchase seventy-five additional prints of the Bruegel from some other manufacturer. But his failure or inability to do so does not bar him from any other remedy open to him.

Sue for Damages for Nondelivery

Bunker could sue for damages for nondelivery. Under Section 2-713 of the UCC, the measure of damages is the difference between the market price at the time when the buyer learned of the breach and the contract price (plus incidental damages, less expenses saved). Suppose Bunker could have bought seventy-five prints for $125 on the day Howard called to say he would not be sending the rest of the order. Bunker would be entitled to $1,875—the market price ($9,375) less the contract price ($7,500). This remedy is available even if he did not in fact purchase the substitute prints. Suppose that at the time of breach, the original painting was worth $15,000 (Howard having just sold it to someone else at that price). Bunker would be entitled to an additional $5,000, which would be the difference between his contract price and the market price.

For leases, the UCC, Section 2A-519(1), provides the following: “the measure of damages for non-delivery or repudiation by the lessor or for rejection or revocation of acceptance by the lessee is the present value, as of the date of the default, of the then market rent minus the present value as of the same date of the original rent, computed for the remaining lease term of the original lease agreement, together with incidental and consequential damages, less expenses saved in consequence of the lessor’s default.”

Recover the Goods

If the goods are unique—as in the case of the original Bruegel—Bunker is entitled to specific performance—that is, recovery of the painting. This section is designed to give the buyer rights comparable to the seller’s right to the price and modifies the old common-law requirement that courts will not order specific performance except for unique goods. It permits specific performance “in other proper circumstances,” and these might include particular goods contemplated under output or requirements contracts or those peculiarly available from one market source.Uniform Commercial Code, Sections 2-716(1) and 2A-521(1).

Even if the goods are not unique, the buyer is entitled to replevy them if they are identified to the contract and after good-faith effort he cannot recover them. Replevin is the name of an ancient common-law action for recovering goods that have been unlawfully taken; in effect it is not different from specific performance, and the UCC makes no particular distinction between them in Section 2-716. Section 2A-521 holds the same for leases. In our case, Bunker could replevy the twenty-five prints identified and held by Howard.

Bunker also has the right to recover the goods should it turn out that Howard is insolvent. Under UCC, Section 2-502, if Howard were to become insolvent within ten days of the day on which Bunker pays the first installment of the price due, Bunker would be entitled to recover the original and the prints, as long as he tendered any unpaid portion of the price.

For security interest in goods rightfully rejected, if the buyer rightly rejects nonconforming goods or revokes acceptance, he is entitled to a security interest in any goods in his possession. In other words, Bunker need not return the twenty-five prints he has already received unless Howard reimburses him for any payments made and for any expenses reasonably incurred in their inspection, receipt, transportation, care, and custody. If Howard refuses to reimburse him, Bunker may resell the goods and take from the proceeds the amount to which he is entitled.Uniform Commercial Code, Sections 2-711(3), 2-706, 2A-508(5), and 2A-527(5).

Goods Accepted

The buyer does not have to reject nonconforming goods. She may accept them anyway or may effectively accept them because the time for revocation has expired. In such a case, the buyer is entitled to remedies as long as she notifies the seller of the breach within a reasonable time.Uniform Commercial Code, Sections 2-714(1) and 2A-519(3). In our example, Bunker can receive three types of damages, all of which are outlined here.

Compensatory Damages

Bunker may recover damages for any losses that in the ordinary course of events stem from the seller’s breach. Suppose Howard had used inferior paper that was difficult to detect, and within several weeks of acceptance the prints deteriorated. Bunker is entitled to be reimbursed for the price he paid.

Consequential Damages

Bunker is also entitled to consequential damages.Uniform Commercial Code, Sections 2-714(3), 2-715, and 2A-519(3). These are losses resulting from general or particular requirements of the buyer’s needs, which the seller had reason to know and which the buyer could not reasonably prevent by cover or otherwise. Suppose Bunker is about to make a deal to resell the twenty-five prints that he has accepted, only to discover that Howard used inferior ink that faded quickly. Howard knew that Bunker was in the business of retailing prints and therefore he knew or should have known that one requirement of the goods was that they be printed in long-lasting ink. Because Bunker will lose the resale, he is entitled to the profits he would have made. (If Howard had not wished to take the risk of paying for consequential damages, he could have negotiated a provision limiting or excluding this remedy.) The buyer has the burden or proving consequential damages, but the UCC does not require mathematical precision. Suppose customers come to Bunker’s gallery and sneer at the faded colors. If he can show that he would have sold the prints were it not for the fading ink (perhaps by showing that he had sold Bruegels in the past), he would be entitled to recover a reasonable estimate of his lost profits.

In De La Hoya v. Slim’s Gun Shop the plaintiff purchased a handgun from the defendant, a properly licensed dealer. While the plaintiff was using it for target shooting, he was questioned by a police officer, who traced the serial number of the weapon and determined that—unknown to either the plaintiff or the defendant—it had been stolen. The plaintiff was arrested for possession of stolen property and incurred, in 2010 dollars, $3,000 in attorney fees to extricate himself from the criminal charges. He sued the defendant for breach of the implied warranty of title and was awarded the amount of the attorney fees as consequential damages. On appeal the California court held it foreseeable that the plaintiff would get arrested for possessing a stolen gun, and “once the foreseeability of the arrest is established, a natural and usual consequence is that the [plaintiff] would incur attorney’s fee.”De La Hoya v. Slim’s Gun Shop, 146 Cal. Rptr. 68 (Super. 1978). Compare with In re Stem in the exercises later in this chapter.

Incidental Damages

Section 2-715 of the UCC allows incidental damages, which are “damages resulting from the seller’s breach including expenses reasonably incurred in inspection, receipt, transportation and care and custody of goods rightfully rejected, any commercially reasonable charges, expenses or commissions in connection with effecting cover and any other reasonable expense incident to the delay or other breach.” Section 2A-520(1) of the UCC is similar for leases.

KEY TAKEAWAY

Parties to a contract for the sale of goods may specify what the remedies will be in case of breach. They may limit or exclude remedies, but the UCC insists that there be some remedies; if the parties agree to liquidated damages, the amount set cannot be a penalty.

If the parties do not agree to different remedies for the seller in case the buyer defaults, the UCC sets out remedies. As to the seller’s obligation, he may cancel the contract. As to the goods, he may withhold or stop delivery, identify conforming goods to the contract, or reclaim goods upon the buyer’s insolvency. As to money, he may resell and recover damages or lost profits and recover the price. Unless they are inconsistent, these remedies are cumulative. The point of the range of remedies is, as much as possible, to put the nonbreaching seller in the position she would have been in had there been no breach. The aggrieved lessor is entitled to similar remedies as the seller.

The UCC also provides a full panoply of remedies available to a buyer if the seller fails to deliver goods or if the buyer rightfully rejects them or revokes her acceptance. As to the buyer’s obligations, she may cancel the contract. As to the goods, she may claim a security interest in those rightfully rejected, recover goods identified if the seller is insolvent, or replevy or seek specific performance to get goods wrongfully withheld. As to money, she may recover payments made or cover and recover damages for nondelivery. If the buyer accepts nonconforming goods, she is entitled to damages for breach of warranty. These remedies are cumulative, so the aggrieved buyer may pursue any of them, unless the remedies are mutually exclusive. The Article on leases provides basically the same remedies for the aggrieved lessee (UCC 2A 520–523).

EXERCISES

  1. What are the four things a breaching seller could do to cause the buyer grief, commercially speaking?
  2. If the buyer breaches, what rights does the seller have in regard to the goods?
  3. In regard to the money owed to her?
  4. In regard to the continued existence of the contract?
  5. What are the four things a breaching buyer could do to cause the seller grief, commercially speaking?
  6. If the seller breaches, what rights does the buyer have in regard to the goods?
  7. In regard to the money owed to him?
  8. In regard to the continued existence of the contract?

19.4 Excuses for Nonperformance

LEARNING OBJECTIVES

  1. Recognize how parties are discharged if the goods are destroyed.
  2. Determine what defenses are valid when it becomes very difficult or impossible to perform.
  3. Understand the UCC’s position on the right to adequate assurances and anticipatory repudiation.

In contracts for the sale of goods, as in common law, things can go wrong. What then?

Casualty to Identified Goods

As always, the parties may agree what happens if the goods are destroyed before delivery. The default is Sections 2-613 and 2A-221(a) of the Uniform Commercial Code (UCC). The UCC says that “where the contract requires for its performance goods identified when the contract is made, and the goods suffer casualty without fault of either party before the risk of loss passes to the buyer,…then (a) if the loss is total the contract is avoided; and (b) if the loss is partial the buyer may nevertheless accept them with due allowance for the goods’ defects.” Thus if Howard ships the original Bruegel to Bunker but the painting is destroyed, through no fault of either party, before delivery occurs, the parties are discharged. If the frame is damaged, Bunker could, if he wants, take the painting anyway, but at a discount.

The UCC’s Take on Issues Affecting “Impossibility”

Although this matter was touched on in Chapter 15 "Discharge of Obligations", it is appropriate to mention briefly again the UCC’s treatment of variations on the theme of “impossibility.”

Impracticability

Sections 2-614(1) and 2A-404(1) of the UCC require reasonable substitution for berthing, loading, and unloading facilities that become unavailable. They also require reasonable substitution for transportation and delivery systems that become “commercially impracticable”; if a practical alternative exists, “performance must be tendered and accepted.” If Howard agreed to send the prints by rail, but a critical railroad bridge is unusable and no trains can run, delivery by truck would be required.

Section 2-615 of the UCC says that the failure to deliver goods is not a breach of the seller’s duty “if performance as agreed has become impracticable by the occurrence of a contingency the non-occurrence of which was a basic assumption on which the contract was made or by compliance in good faith with any applicable foreign or domestic government regulation or order whether or not it later proves to be invalid.” Section 2A-405(b) of the UCC is similar for leases.

The CISG provides something similar at Article 79: “A party is not liable for a failure to perform any of his obligations if he proves that the failure was due to an impediment beyond his control and that he could not reasonably be expected to have taken the impediment into account at the time of the conclusion of the contract or to have avoided or overcome it or its consequences.”

Right to Adequate Assurances of Performance

Section 2-609, Comment 1, of the UCC observes that “the essential purpose of a contract…is actual performance [but] a continuing sense of reliance and security that the promised performance will be forthcoming when due is an important feature of the bargain.” Thus the UCC says that if one party has “reasonable grounds for insecurity arise…either party may in writing demand adequate assurance and until he receives such assurance may if commercially reasonable suspend [his own] performance[.]”

The CISG has a similar take at Article 71: “A party may suspend the performance of his obligations if, after the conclusion of the contract, it becomes apparent that the other party will not perform a substantial part of his obligations. A party suspending performance, whether before or after dispatch of the goods, must immediately give notice of the suspension to the other party and must continue with performance if the other party provides adequate assurance of his performance.”

Anticipatory Repudiation

Obviously if a person repudiates the contract it’s clear she will not perform, but what if she repudiates before time for performance is due? Does the other side have to wait until nonperformance actually happens, or can he sue in anticipation of the other’s default? Sections 2-610 and 2A-402 of the UCC say the aggrieved party can do either: wait for performance or “resort to any remedy for breach.” Under the UCC, Sections 2-611 and 2A-403, the one who has anticipatorily repudiated can “retract his repudiation unless the aggrieved party has since the repudiation cancelled or materially changed his position[.]”

Suppose that Howard has cause to suspect that if he does deliver the goods, Bunker won’t pay. Howard may write to Bunker and demand—not request—assurances of adequate performance. If such assurances are not adequately forthcoming, Howard may assume that Bunker has repudiated the contract and have remedies.

Article 72 of the CISG is pretty much the same: “If prior to the date for performance of the contract it is clear that one of the parties will commit a fundamental breach of contract, the other party may declare the contract avoided.”

KEY TAKEAWAY

If, through no fault of either party, the goods are destroyed before the risk of loss has passed from the seller to the buyer, the parties are both discharged. If the expected means of performance is impossible, but an alternative is available, the alternative must be utilized. If performance becomes impracticable because of an unexpected contingency, failure to deliver the goods is excused. But a party who has concerns whether the other side will perform is entitled to adequate assurances of performance; if they are not forthcoming, the worried party may suspend performance. Where a party repudiates a contract before performance is due, the other side may sue immediately (anticipatory repudiation) or may wait until the time performance comes due and then sue.

EXERCISES

  1. Suppose Plaintiff sues Defendant for breach of contract, and Defendant successfully raises an excuse for nonperformance. What liability does Defendant have now?
  2. The contract read that the goods would be “shipped F.O.B. Seattle, by Burlington Northern Rail to the buyer in Vancouver, B.C.” Due to heavy rain and mudslides, the rail line between Seattle and points north was impassable. Buyer insists Seller is obligated to send the goods by motor truck; Seller insists her performance has become impossible or at least that shipment must await the rail-line clearance. Who is correct? Explain.
  3. Buyer manufactured ceramic insulators and ordered the dies into which the liquid ceramic would be poured for hardening and finishing from Seller, to be delivered April 15. The first test batch of a dozen dies arrived on February 15; these dies were defective. Buyer wrote inquiring whether the defects could be remedied in time for the final delivery. Seller responded, “We are working to address the problems here.” Buyer again inquired; Seller responded, “As I said, we are working on the problems.” Buyer fretted that the deadline—two months in the future—would not be met. What remedy, if any, does Buyer have now?

19.5 Cases

Limitations of Remedy Results in No Remedy

Hartzell v. Justus Co., Inc.

693 F.2d 770 (8th Cir. S.D. 1982)

Arnold, J.

This is a diversity case arising out of the purchase by Dr. Allan Hartzell of Sioux Falls, South Dakota, of a log home construction kit manufactured by the defendant Justus Homes. Dr. Hartzell purchased the package in 1977 for $38,622 [about $135,000 in 2010 dollars] from Del Carter, who was Justus Homes’ dealer for the Sioux Falls area. He also hired Carter’s construction company, Natural Wood Homes, to build the house. Hartzell, who testified that the home eventually cost about $150,000, was dissatisfied with the house in many respects. His chief complaints were that knotholes in the walls and ceiling leaked rain profusely, and that the home was not weather tight because flashings were not included in the roofing materials and because the timbers were not kiln-dried and therefore shrank. He also complained that an undersized support beam, which eventually cracked, was included in the package. This latter defect was alleged to have resulted in cracks in the floor and inside doors that would not close. Hartzell further alleged that these structural defects were only partially remediable, and that the fair market value of the house was reduced even after all practicable repairs had been made. Alleging breach of implied and express warranties and negligence, he sought damages for this loss in value and for the cost of repairs. After a two-day trial, the jury returned a plaintiff’s verdict for $34,794.67.

Justus Homes contends the District Court erred in failing to instruct the jury on a limitation-of-remedies clause contained in its contract with the plaintiff. The defendants rely on Clause 10c of the contract, which says Justus will repair or replace defective materials, and Clause 10d, which states that this limited repair or replacement clause is the exclusive remedy available against Justus [emphasis added]. These agreements, Justus asserts, are valid under the Uniform Commercial Code 2-719(1). Section 2-719(1) states:

(1) Subject to the provisions of subsections (2) and (3) of this section and of § 57A-2-718 on liquidation and limitation of damages,

(a) The agreement may provide for remedies in addition to or in substitution for those provided in this chapter and may limit or alter the measure of damages recoverable under this chapter, as by limiting the buyer’s remedies to return of the goods and repayment of the price or to repair and replacement of nonconforming goods or parts; and

(b) Resort to a remedy as provided is optional unless the remedy is expressly agreed to be exclusive, in which case it is the sole remedy.

Subsection (1) of section 2-719 is qualified by subsection (2): “Where circumstances cause an exclusive or limited remedy to fail of its essential purpose, remedy may be had as provided in this title.”…

The jury’s verdict for the plaintiff in an amount almost exactly equal to the plaintiff’s evidence of cost of repairs plus diminution in market value means it must have found that the structural defects were not entirely remediable. Such a finding necessarily means that the limited warranty failed of its essential purpose.

Two of our recent cases support this conclusion. In Soo Line R.R. v. Fruehauf Corp., 547 F.2d 1365 (8th Cir.1977), the defendant claimed, relying on a limitation-of-remedies clause similar to the one involved here, that the plaintiff’s damages should be limited to the reasonable cost of repairing the railroad cars that plaintiff had bought from defendant. The jury verdict included, among other things, an award for the difference between the value of the cars as actually manufactured, and what they would have been worth if they had measured up to the defendant’s representations. This Court affirmed the verdict for the larger amount. We held, construing the Minnesota U.C.C., which is identical to § 2-719 as adopted in South Dakota, that the limitation-of-remedies clause was ineffective because the remedy as thus limited failed of its essential purpose. The defendant, though called upon to make the necessary repairs, had refused to do so, and the repairs as performed by the plaintiff itself “did not fully restore the cars to totally acceptable operating conditions.”

Here, Justus Homes attempted to help with the necessary repairs, which is more than Fruehauf did in the Soo Line case, but after the repairs had been completed the house was still, according to the jury verdict, not what Justus had promised it would be. The purpose of a remedy is to give to a buyer what the seller promised him—that is, a house that did not leak. If repairs alone do not achieve that end, then to limit the buyer’s remedy to repair would cause that remedy to fail of its essential purpose.…

An analogous case is Select Pork, Inc. v. Babcock Swine, Inc. [Citation], applying § 2-719 as adopted in Iowa. The defendant had promised to deliver to plaintiff certain extraordinary pigs known as Midwestern Gilts and Meatline Boars. Instead, only ordinary pigs were delivered. Plaintiff sued for breach of warranty, and defendant claimed that its damages, if any, should be limited to a return of the purchase price by an express clause to that effect in the contract. The District Court held that the clause was unenforceable because it was unconscionable, see § 2-719(3), and because it failed of its essential purpose. We affirmed,…“Having failed to deliver the highly-touted special pigs, defendants may not now assert a favorable clause to limit their liability.” So here, where the house sold was found by the jury to fall short of the seller’s promises, and where repairs could not make it right, defendant’s liability cannot be limited to the cost of repairs. If the repairs had been adequate to restore the house to its promised condition, and if Dr. Hartzell had claimed additional consequential damages, for example, water damage to a rug from the leaky roof, the limitation-of-remedies clause would have been effective. But that is not this case.

There was no double recovery here: the verdict was not for cost of repair plus the entire decrease in market value, but rather for cost of repair plus the decrease in market value that still existed after all the repairs had been completed.

[T]he evidence in the record all demonstrate[s] that the repair or replacement clause was a failure under the circumstances of this case. Some of the house’s many problems simply could not be remedied by repair or replacement. The clause having failed of its essential purpose, that is, effective enjoyment of implied and express warranties, the plaintiff was entitled, under UCC § 2-719(2), to any of the buyer’s remedies provided by the Code. Among these remedies are consequential damages as provided in §§ 2-714 and 2-715(2).…

The judgment is affirmed.

CASE QUESTIONS

  1. What did the seller here limit itself to do in case of defects? What was the limitation of remedy?
  2. Did Justus Homes disclaim implied and expressed warranties with its contract language regarding limitation of remedies?
  3. Was the essential purpose of the limitation of remedy to protect the party benefiting from it—here, the seller of the log home kit—or was the essential purpose of the limitation of remedy, as the court said, “effective enjoyment of implied and expressed warranties”?
  4. In a part of the opinion excised, the court wrote, “A finding of unconscionability is, as a matter of logic, simply unnecessary in cases where § 2-719(2) applies.” Would it be easier simply to say that the limitation of liability here was unconscionable?

Cure for Improper Delivery

Wilson v. Scampoli

228 A.2d 848 (D.C. App. 1967)

Myers, J.

This is an appeal from an order of the trial court granting rescission of a sales contract for a color television set and directing the return of the purchase price plus interest and costs.

Appellee [Mrs. Kolley’s father] purchased the set in question on November 4, 1965, paying the total purchase price in cash. The transaction was evidenced by a sales ticket showing the price paid and guaranteeing ninety days’ free service and replacement of any defective tube and parts for a period of one year. Two days after purchase the set was delivered and uncrated, the antennae adjusted and the set plugged into an electrical outlet to “cook out.” When the set was turned on however, it did not function properly, the picture having a reddish tinge. Appellant’s delivery man advised the buyer’s daughter, Mrs. Kolley, that it was not his duty to tune in or adjust the color but that a service representative would shortly call at her house for that purpose. After the departure of the delivery men, Mrs. Kolley unplugged the set and did not use it.

On November 8, 1965, a service representative arrived, and after spending an hour in an effort to eliminate the red cast from the picture advised Mrs. Kolley that he would have to remove the chassis from the cabinet and take it to the shop as he could not determine the cause of the difficulty from his examination at the house. He also made a written memorandum of his service call, noting that the television ‘\”Needs Shop Work (Red Screen).” Mrs. Kolley refused to allow the chassis to be removed, asserting she did not want a ‘repaired’ set but another ‘brand new’ set. Later she demanded the return of the purchase price, although retaining the set. Appellant refused to refund the purchase price, but renewed his offer to adjust, repair, or, if the set could not be made to function properly, to replace it. Ultimately, appellee instituted this suit against appellant seeking a refund of the purchase price. After a trial, the court ruled that “under the facts and circumstances the complaint is justified. Under the equity powers of the Court I will order the parties put back in their original status, let the $675 [about $4500 in 2010 dollars] be returned, and the set returned to the defendant.”

Appellant does not contest the jurisdiction of the trial court to order rescission in a proper case, but contends the trial judge erred in holding that rescission here was appropriate. He argues that he was always willing to comply with the terms of the sale either by correcting the malfunction by minor repairs or, in the event the set could not be made thereby properly operative, by replacement; that as he was denied the opportunity to try to correct the difficulty, he did not breach the contract of sale or any warranty thereunder, expressed or implied.

[The District of Columbia UCC 2-508] provides:

(1) Where any tender or delivery by the seller is rejected because non-conforming and the time for performance has not yet expired, the seller may seasonably notify the buyer of his intention to cure and may then within the contract time make a conforming delivery.

(2) Where the buyer rejects a nonconforming tender which the seller had reasonable grounds to believe would be acceptable with or without money allowance the seller may if he seasonably notifies the buyer have a further reasonable time to substitute a conforming tender.

Removal of a television chassis for a short period of time in order to determine the cause of color malfunction and ascertain the extent of adjustment or correction needed to effect full operational efficiency presents no great inconvenience to the buyer. In the instant case, appellant’s expert witness testified that this was not infrequently necessary with new televisions. Should the set be defective in workmanship or parts, the loss would be upon the manufacturer who warranted it free from mechanical defect. Here the adamant refusal of Mrs. Kolley, acting on behalf of appellee, to allow inspection essential to the determination of the cause of the excessive red tinge to the picture defeated any effort by the seller to provide timely repair or even replacement of the set if the difficulty could not be corrected. The cause of the defect might have been minor and easily adjusted or it may have been substantial and required replacement by another new set—but the seller was never given an adequate opportunity to make a determination.

We do not hold that appellant has no liability to appellee, but as he was denied access and a reasonable opportunity to repair, appellee has not shown a breach of warranty entitling him either to a brand new set or to rescission. We therefore reverse the judgment of the trial court granting rescission and directing the return of the purchase price of the set.

Reversed.

CASE QUESTIONS

  1. Why did the seller “have reasonable grounds to believe [the television] would be acceptable”?
  2. What did Mrs. Kolley want?
  3. Does this case require a buyer to accept patchwork goods or substantially repaired articles in lieu of flawless merchandise?

Seller’s Remedies When Buyer Defaults

Santos v. DeBellis

901 N.Y.S.2d 457 (N.Y. Sup.App. 2010)

Molia, J.

On March 1, 2008 and March 11, 2008, plaintiff made payments to defendant of $3,000 each, in connection with the purchase of a mobile home located in Fort Pierce, Florida. Thereafter, on March 13, 2008, plaintiff and defendant signed an agreement which had been prepared by defendant. The agreement described the subject property by its location, recorded the fact that plaintiff had paid defendant deposits totaling $6,000, set forth a closing date of March 25, 2008, and specified that “the remaining $27,000.00” was payable at closing to defendant by a guaranteed financial instrument. Plaintiff never paid the outstanding balance and brought this action to recover the $6,000 deposit she paid to defendant. Following a nonjury trial, judgment was awarded in favor of defendant dismissing the complaint.

Because the sale of a mobile home constitutes a contract for the sale of goods rather than of real property [Citations], the parties’ agreement was governed by the Uniform Commercial Code. The agreement, which was made after plaintiff had made the two $3,000 “deposit” payments, constituted a memorandum in confirmation of an oral agreement and, even though it omitted some terms, was sufficient to satisfy the statute of frauds [Citations].

Section 2-718 of the Uniform Commercial Code specifies that in the absence of a contractual provision with respect to the liquidation or limitation of damages and the return of deposits,

(2) Where the seller justifiably withholds delivery of goods because of the buyer’s breach, the buyer is entitled to restitution of any amount by which the sum of his payments exceeds…

(b) [in the absence of contractually fixed terms] twenty per cent of the value of the total performance for which the buyer is obligated under the contract or $500, whichever is smaller.

(3) The buyer’s right to restitution under subsection (2) is subject to offset to the extent that the seller establishes

(a) a right to recover damages under the provisions of this Article other than subsection (1), and

(b) the amount or value of any benefits received by the buyer directly or indirectly by reason of the contract.

Here, notwithstanding the fact that plaintiff, as buyer, had breached the contract, defendant failed to demonstrate any damages resulting therefrom; nor did defendant establish that plaintiff had received any benefits directly or indirectly by reason of the parties’ agreement (see UCC 2-718[3]). Therefore, pursuant to UCC 2-718(2), plaintiff was entitled to the return of all but $500 of her deposit.

The order of the District Court dismissing the complaint is accordingly reversed, and judgment is awarded to plaintiff in the principal sum of $5,500.

CASE QUESTIONS

  1. If the plaintiff had been a dealer in mobile homes and the unit here had been part of his inventory, he would be entitled to claim lost profits on the sale of one unit. Here, apparently, the plaintiff seller was a private party. Why was he not entitled to any damages greater than $500?
  2. New York adopted the UCC in 1964. Five hundred dollars in 1964 would be worth about $3,500 in 2010. Why isn’t the change in the dollar’s value recognized here?

Buyer’s Remedies When Seller Breaches

[Note: this case is slightly edited by the authors.]

Furlong v. Alpha Chi Omega Sorority

657 N.E.2d 866 (Ohio Mun. 1993)

Bachman, J.

In late September through mid-October 1992, plaintiff Johnathan James Furlong (“Furlong”) contacted defendant Alpha Chi Omega Sorority (“AXO”), by phoning the chairperson of its social committee, Emily Lieberman (“Emily”), between a dozen and a dozen and a half times.

Ultimately (about the first week in October), Furlong received Emily’s order for one hundred sixty-eight imprinted sweaters at $21.50 each (plus one free sweater) for delivery on Friday, October 23, 1992, so as to arrive in time for AXO’s Midnight Masquerade III on the evening of Saturday, October 24, 1992.

The price was to be $3,612, [about $5600 in 2010 dollars] payable as follows: $2,000 down payment when the contract was made, and $1,612 balance when the sweaters were delivered.

An oral contract for the sale of goods (the imprinted sweaters) was made between Furlong and AXO, at a definite price and with specified dates for payment and for delivery.

At some point in those phone calls with Furlong, Emily said that the sweaters were to be custom designed with the following specified design: namely, with three colors (hunter green letters on top of maroon letters outlined in navy blue, and hunter green masks). Furlong promised to have them so imprinted (by a third party whom he would select).…Thereafter, he delivered to Emily an Ohio Wesleyan sweater with maroon letters to show her the maroon color.…Additionally, he faxed to Emily a two-page description of the sweaters, which not only included the designs for the fronts and the backs of the sweaters, but also included arrows showing where each of the three colors would go (hunter green letters on top of maroon letters outlined in navy blue, and hunter green masks).

Furlong and Emily created an express warranty by each of the above three statutory means: namely, by affirmation of fact (his initial phone calls); by sample (the maroon sweater) by description (the fax).This express warranty became part of the contract. Each of the three methods of showing the express warranty was not in conflict with the other two methods, and thus they are consistent and cumulative, and constitute the warranty. [2-317]

The design was a “dickered” aspect of the individual bargain and went clearly to the essence of that. Thus, the express warranty was that the sweaters would be in accordance with the above design (including types of colors for the letters and the mask, and the number of colors for the same). Further, the express warranty became part of the contract.

On October 13, 1992, AXO mailed Furlong a $2,000 check for the down payment; he deposited it in his bank account on October 16, 1992. Thereafter, as discussed below, Furlong had the sweaters imprinted (on Thursday, October 22) and delivered to AXO (on Friday, October 23). Upon receipt of the delivery, AXO gave a check to Furlong’s agent in the amount of $1,612 for the balance of the purchase price. However, later on that day, AXO inspected the sweaters, discovered the design changes (mentioned below), caused AXO’s bank to stop payment on the check, and stated AXO’s objections in a phone call with Furlong. AXO has never paid Furlong that balance on the purchase price.

Furlong’s obligation as the seller was to transfer and deliver the goods in accordance with the contract. AXO’s obligation was to accept and pay in accordance with that contract. [2-301] We will now discuss whether it legally did so.

Furlong was a jobber for Argento Bros., Inc. (“Argento”) and had Argento print the sweaters. In doing so, Furlong worked with Argento’s artists. Early in the morning of Thursday (October 22, 1992), the artist(s) began to prepare the art work and recommended changes to the design. Furlong authorized the artist(s) to change the design without the knowledge or consent of AXO. Argento spent about eight hours printing the sweaters all day Thursday. Furlong did not phone AXO about the changes until the next day, Friday (October 23), after the sweaters were printed with those changes. Here are the five design changes that he made:

  • The first change was to delete the agreed-upon outline for the letters (namely, the navy blue outline).
  • The second change was to reduce the agreed-upon number of colors for the fronts and the backs (from three colors per side to two colors per side).
  • The third change was to alter one of the agreed-upon colors (from maroon to red).
  • The fourth change was to alter the agreed-upon scheme of colors for the letters on the fronts and the backs (namely, both sides were to have the same two colors of maroon and hunter green; whereas in fact the backs had neither of those colors, and instead had a navy blue color for the letters).
  • The fifth change was to alter the agreed-upon color of the masks (from hunter green to maroon—actually red).

The court specifically finds that the color was red (actually, scarlet) and was not maroon (like the maroon-colored letters on the Ohio Wesleyan sweater).

The sweaters did not conform to the contract (specifically, the express warranty in the contract). Thus (in the words of the statute), the sweaters did “fail in any respect to conform to the contract.” Actually, the sweaters failed in at least five respects. [2-601] Further, not only did they “fail in any respect,” they failed in a substantial respect. In either event, they were a nonconforming tender of goods. [2-601]

On Friday morning (October 23), Furlong picked up the five to six boxes of sweaters from Argento and had a friend deliver them from Columbus to Bowling Green. The boxes arrived at the AXO house around midday. Sometime thereafter on the same day, Emily inspected one of them and screamed her dismay upon discovering that the sweaters were not what AXO had ordered.

The court rejects Furlong’s assertion that he did all that he could do under the circumstances. The obvious answer is that he did not do enough. He should have gotten AXO’s prior consent to the changes. He could have done this by providing for more lead time-between the time that Argento prepared the art work and the time that it printed the sweaters. Instead, he had both done at the same time (Thursday morning).

Finally, and alternatively, plaintiff should have entered into a contract that gave him discretion to make design changes without AXO’s consent. We must remember that “these sweaters,” as Furlong himself admits (and describes), were to be “custom-designed” for AXO. Thus, they were to be printed according to AXO’s specifications, and not according to Furlong’s discretion.

Next, Furlong asserts that AXO—after learning of the changes—should have agreed to his offer of compromise: namely, that he would reduce the unit price of the sweaters in exchange for AXO’s keeping them and paying the reduced price. Also, Furlong asserts that AXO should have communicated his compromise offer to AXO’s members and pledges. In both respects, the court disagrees: Although the law allowed AXO to do so, it did not require AXO to do. Instead, AXO did exactly what the law allowed: AXO rejected the nconforming goods in whole.

About 4:00 p.m. on the same day that the sweaters arrived at the AXO house (Friday, October 23), Amy—as the AXO president—phoned Furlong. She said that the sweaters were not what AXO had ordered. She stated the specifics as to why the sweaters were not as ordered. She offered to return the sweaters to him, but he said “No.” AXO still has possession or custody of the boxes of sweaters.

[The UCC] provides: “Rejection of goods must be within a reasonable time after their delivery * * *. It is ineffective unless the buyer seasonably notifies the seller.” [2-602] AXO did what this statute requires.

That statute further provides: “[I]f the buyer has before rejection taken physical possession of goods * * *, he is under a duty after rejection to hold them with reasonable care at the seller’s disposition for a time sufficient to permit the seller to remove them[.]” [2-602(2)(b)] AXO has done this, too. From the above, it is seen that AXO legally rejected the sweaters on the same day that AXO received physical possession of them.

The court disagrees with Furlong’s assertion that AXO accepted the sweaters. He is confusing a layman’s understanding of the term accept (“to receive a thing [with a consenting mind]),” Webster’s Collegiate Dictionary (5 Ed.1947), at 6, with the statutory meaning of the term. The mere fact that AXO took physical possession of the sweaters does not, by itself, mean that AXO legally “accepted” them.

In regard to…seller’s remedies, Furlong has no legal remedies because AXO did not breach the contract. Thus, he is not entitled to an award for the $1,612 balance that he claims is due on the contract price.

As concluded above, AXO rightfully rejected the sweaters, after having paid part of the purchase price: namely, $2,000. AXO is entitled to cancel the contract and to recover the partial payment of the purchase price. [2-606]

Also, as concluded above, AXO still has rightful possession or control of the sweaters. AXO has a security interest in the sweaters in its possession or control for the part payment made on the purchase price—but when reimbursed for that part payment AXO must return the sweaters to Furlong.

The court will prepare, file, and serve a judgment entry as follows: dismissing with prejudice Furlong’s claim against all defendants; dismissing with prejudice Emily Lieberman’s and Amy Altomondo’s counterclaims against Furlong; granting AXO’s counterclaim (for $2,000, plus ten percent per annum postjudgment interest and costs).

Further, that entry will order AXO’s attorney (Mr. Reddin) to retain possession of the sweaters either until further court order or until AXO’s judgment is satisfied in full (whereupon he shall surrender the sweaters to Furlong if Furlong picks them up within thirty days thereafter, or, if Furlong does not, he may then dispose of them as abandoned property without any liability).

Judgment accordingly.

CASE QUESTIONS

  1. Surely the plaintiff could not have thought that the radically altered design would be acceptable for the young women’s masquerade ball. On what basis did he think he would be entitled to the full payment contracted for?
  2. Whether Amy Altomondo knew it or not, she did what the UCC says a buyer should do when nonconforming goods are delivered. What are those steps?
  3. What does it mean that AXO has a security interest in the sweaters? Security for what?

19.6 Summary and Exercises

Summary

As with most of the Uniform Commercial Code (UCC), the parties may specify the terms of their performance. Only if they fail to do so does Article 2 (and 2A) provide the terms for them. The seller’s duty is to make a timely delivery of conforming goods. In the absence of agreement, the time for delivery is a reasonable one, and the place of delivery is the seller’s place of business. All goods must be tendered in a single delivery, unless circumstances permit either party the right to make or demand delivery in lots.

If the seller ships nonconforming goods but has time to meet his contractual obligations or if he reasonably believed the goods would be suitable, he may notify the buyer of his intention to cure, and if he does so in a timely manner the buyer must pay.

The buyer’s general obligation is to inspect, accept, and pay. If an inspection reveals that the goods are nonconforming, the buyer may reject them; if he has accepted because defects were latent or because he received assurances that the defects would be cured, and they are not, the buyer may revoke his acceptance. He then has some duties concerning the goods in his possession. The buyer must pay for any conforming goods; payment may be in any manner consistent with current business customs. Payment is due at the time and place at which the buyer will ultimately receive the goods.

The general policy of the UCC is to put an aggrieved party in as good a position as she would have been had the other party fully performed. The parties may specify or limit certain remedies, but they may not eliminate all remedies for a breach. However, if circumstances make an agreed-on remedy inadequate, then the UCC’s other remedies apply; parties may not unconscionably limit consequential damages; they may agree to liquidated damages, but not to unreasonable penalties.

In general, the seller may pursue the following remedies: withhold further delivery, stop delivery, identify to the contract goods in her possession, resell the goods, recover damages or the price, or cancel the contract. In addition, when it becomes apparent that the buyer is insolvent, the seller may, within certain time periods, refuse to deliver the remaining goods or reclaim goods already delivered.

The buyer, in general, has remedies. For goods not yet received, she may cancel the contract; recover the price paid; cover the goods and recover damages for the difference in price; or recover the specific goods if they are unique or in “other proper circumstances.” For goods received and accepted, the buyer may recover ordinary damages for losses that stem from the breach and consequential damages if the seller knew of the buyer’s particular needs and the buyer could not reasonably cover.

The UCC provides some excuses for nonperformance: casualty of the goods, through no fault of either party; the nonhappening of presupposed conditions that were a basic assumption of the contract; substituted performance if the agreed-on methods of performance become impracticable; right to adequate assurances of performance when reasonable grounds for insecurity of performance arise; anticipatory repudiation and resort to any remedy, before time for performance is due, is allowed if either party indicates an unwillingness to perform.

EXERCISES

  1. Anne contracted to sell one hundred cans of yellow tennis balls to Chris, with a delivery to be made by June 15.

    1. On June 8, Anne delivered one hundred cans of white tennis balls, which were rejected by Chris. What course of action would you recommend for Anne, and why?
    2. Assume Ann had delivered the one hundred cans of white balls on June 15; these were rejected by Chris. Under what circumstances might Anne be allowed additional time to perform the contract?
    3. If the contract did not specify delivery, when must Anne deliver the tennis balls?
  2.  

    1. When Anne delivers the tennis balls, does Chris have a right to inspect them? If Chris accepts the white tennis balls, may the acceptance be revoked?
    2. Assume Chris decided she could use twenty-five cans of the white balls. Could she accept twenty-five cans and reject the rest?
    3. Suppose Anne delivered white tennis balls because a fire at her warehouse destroyed her entire stock of yellow balls. Does the fire discharge Anne’s contractual duties?
    4. If Chris rejected the white tennis balls and Anne refused to deliver yellow ones, may Chris recover damages? If so, how would they be calculated?
  3. In 1961, Dorothy and John Wilson purchased a painting from Hammer Galleries titled Femme Debout. It cost $11,000 (about $78,000 in 2010 dollars) and came with this promise: “The authenticity of this picture is guaranteed.” In 1984, an expert deemed the painting a fake. The district court held that the Wilsons’ suit for breach of warranty, filed in February 1987—twenty-one years after its purchase—was barred by the UCC’s four-year statute of limitations. The Wilsons argued, however, that the Code’s exception to the four-year rule applied:Uniform Commercial Code, Section 2-725(2). “A breach of warranty occurs when tender of delivery is made, except where a warranty explicitly extends to future performance and discovery must await the time of such performance the cause of action accrues when the breach is or should have been discovered.”

    They said the painting “performed” by being an authentic Vuillard—a French artist—and that the warranty of authenticity not only guaranteed the present “being” of the painting but also extended, as required by 2-725(2), to the future existence as a Vuillard. Therefore, they contended, explicit words warranting future performance would be superfluous: a warranty that promises authenticity “now and at all times in the future” would be redundant. How should the court rule?

  4. Speedi Lubrication Centers Inc. and Atlas Match Corp. entered into a contract that provided for Speedi to buy 400,000 advertising matchbooks from Atlas, to be paid for within thirty days of delivery of each shipment. Orders for such matches required artwork, artists’ commissions, and printing plates. Atlas sent twenty-two cases of matches to Speedi with an invoice showing $2,100 owed. Almost ninety days later, Speedi sent Atlas a check for $1,000, received the same day Atlas sent Speedi a letter declaring Speedi to be in material breach of the contract. A second check for $1,100 was later received; it bounced but was later replaced by a cashier’s check. The contract provided that an untimely payment was a breach, and it included these provisions related to liquidated damages:

    Atlas shall have the right to recover from Purchaser the price of all matchbooks and packaging delivered and/or identified to this agreement at the time of Purchaser’s breach hereof and shall be additionally entitled to recover fifty percent (50%) of the contract price of matchbooks and/or packaging ordered hereby, but not delivered or identified to this Agreement at the time of Purchaser’s breach. Purchaser agrees that the percentage as specified hereinabove…will be reasonable and just compensation for such breach, and Purchaser hereby promises to pay such sum as liquidated damages, not as penalty in the event of any such breach.

    On appeal, Speedi complained that the liquidated damages clause was a penalty. Is the matter settled by the contract saying the liquidated damages are reasonable? On what criteria would a court determine whether liquidated damages are reasonable?

  5. Mrs. Kaiden made a $5,000 deposit on the purchase of new 1973 Rolls-Royce automobile. Lee Oldsmobile, the seller, confirmed the request by transmitting a regular order form, which Mrs. Kaiden signed and returned. The price was $29,500.00 [about $150,000 in 2010 dollars]. Some of the correspondence and a notation on Mrs. Kaiden’s check indicated that delivery was expected in November. The order form, however, specified no delivery date. Further, it contained a disclaimer of liability for delay in delivery beyond the dealer’s control, and it provided that the dealer had the right, upon failure of the purchaser to accept delivery, to retain as liquidated damages any cash deposit made. On November 21, 1973, Mrs. Kaiden notified Lee by telephone that she had purchased another Rolls-Royce elsewhere. She told the salesman to cancel her order. On November 29, Lee Oldsmobile notified Mrs. Kaiden that the car was ready for delivery. She refused delivery and demanded the return of her deposit. The dealer refused. In January 1974, the dealer—without notice to the Kaidens—sold the Rolls-Royce to another purchaser for $26,495. Mrs. Kaiden sued Lee Oldsmobile for the $5,000 deposit. The dealer carefully itemized its losses on the Kaiden deal—$5080.07. On what basis did the court dismiss the liquidated damages clause? What is the consequence of the dealer’s failure to give notice of the private sale under UCC, Section 2-706(3)?
  6. Hemming saw an advertisement for a Cadillac convertible once owned by the famous early rock ’n’ roll singer Elvis Presley. He contracted to buy it from Whitney for $350,000 and sent Whitney $10,000 as a deposit. But, after some delay, Whitney returned the $10,000 and informed Hemming that the car had been sold to another purchaser. What remedy does Hemming have?
  7. Murrey manufactured and sold pool tables. He was approached by Madsen, who had an idea for a kind of electronic pool table that would light up and make sounds like a pinball machine. Madsen made a $70,000 deposit on an order for one hundred tables but then encountered difficulties and notified Murrey that he would be unable to accept delivery of the tables. Murrey broke the tables up, salvaging materials worth about $15,000 and using the rest for firewood. The evidence was that the tables, if completed by Murrey, could have been sold for $45,000 as regular pool tables. Madsen gets his deposit back less expenses incurred by Murrey. But what principle affects Murrey’s measure of damages, his right to claim expenses incurred?
  8. In January 1992, Joseph Perna bought an eleven-year-old Oldsmobile at a New York City police auction sale for $1,800 plus towing fees. It had been impounded by the police for nonpayment of parking tickets. The bill of sale from the police to Perna contained this language: “subject to the terms and conditions of any and all chattel mortgages, rental agreements, liens, conditional bills of sale, and encumbrances that may be on the motor vehicle of the [its original owner].” About a year later Perna sold the car to a coworker, Elio Marino, for $1,200. Marino repaired and improved the car by replacing the radiator, a gasket, and door locks. Ten months after his father bought the car, Marino’s son was stopped by police and arrested for driving a stolen vehicle; Mario paid $600 to a lawyer to get that matter resolved, and he never got the car back from the police. Is Perna liable to Marino for the value of the car? Is Perna liable for the consequential damages—the attorney’s fees? The relevant UCC sections are 2-312(2) and 2-714.
  9. William Stem bought a used BMW from Gary Braden for $6,600 on Braden’s assertion that as far as he knew the car had not been wrecked and it was in good condition. Less than a week later Stem discovered a disconnected plug; when connected the oil-sensor warning light glowed. Mechanics informed Stem that the car was made up of the front end of a 1979 BMW and the rear end of a 1975 BMW, and the front half had 100,000 more miles on it than Stem thought. Six weeks after he purchased the car, Stem wrote Braden a letter that he refused the car and intended to rescind the sale. Braden did not accept return of the car or refund the money, and Braden continued to drive it for seven months and nearly 9,000 miles before suing. He had no other car and needed to transport his child. These issues were before the Alabama Supreme Court, construing UCC, Section 2-608: did Stem’s use of the car, notwithstanding his letter of rescission, constitute such use of it as to be an acceptance? And if not, does Stem owe Braden anything for its use?
  10. Donnelly ordered a leather motorcycle jacket from Leathers Inc. The jacket was specially designed according to Donnelly’s instructions: it had a unique collar, various chromed studs throughout, and buckles, and he required an unusually large size. The coat cost $6,000. Donnelly paid $1,200 as a deposit, but after production was nearly complete, he telephoned Leathers Inc. and repudiated the contract. What should Leathers do now?

SELF-TEST QUESTIONS

  1. In the absence of agreement, the place of delivery is

    1. the buyer’s place of business
    2. the seller’s place of business
    3. either the buyer’s place of business or the buyer’s residence
    4. any of the above
  2. The UCC’s statute of limitations is

    1. two years
    2. three years
    3. four years
    4. none of the above
  3. Under the UCC, if the buyer breaches, the seller can

    1. withhold further delivery
    2. resell the goods still in the seller’s possession
    3. recover damages
    4. do all of the above
  4. If the seller breaches, the buyer can generally

    1. recover the goods, even when the goods have not been identified to the contract and the seller is not insolvent
    2. purchase substitute goods and recover their cost
    3. purchase substitute goods and recover the difference between their cost and the contract price
    4. recover punitive damages
  5. Following a seller’s breach, the buyer can recover the price paid

    1. if the buyer cancels the contract
    2. only for goods the buyer has accepted
    3. for all the goods the buyer was to have received, whether or not they were accepted
    4. under none of the above conditions

SELF-TEST ANSWERS

  1. b
  2. c
  3. d
  4. c
  5. d


    Previous Chapter
    Table of Contents
    Next Chapter

Put the power of the law in your hands